Hi everyone! For this question, I understand how answer choice B wrecks the argument, making it a solid necessary assumption. It was my original answer choice, but I thought I was being tricked at the back end of the test because it seemed like a sufficient assumption. Is it both? Can it be both? If it's not a SA, what am I missing? I hate NA vs SA.
Answer choice B provides the following as a necessary assumption ...
CW-->PBSEKW
The contrapositive of the above conditional plus the author's conclusion make this ...
not(PBSEKW)-->not(CW)
not(PBSEKW)
------------------------------------------------
not(CW)
Seriously, how is this not a SA? It is enough to make the conclusion valid. Please help!
https://classic.7sage.com/lsat_explanations/lsat-68-section-3-question-23/
So fast! Thanks for coming back. I need to review some logic lessons, lol.